Please help timed question (question included in photo). I'll mark as brainiest!

Please Help Timed Question (question Included In Photo). I'll Mark As Brainiest!

Answers

Answer 1

Answer:

Step-by-step explanation:

This is a right triangle trig problem. Picture a right triangle. The height of the triangle is the height of the lighthouse, your distance from the base of the lighthouse is what we are solving for, and the reference angle is 2 degrees, which is the angle of inclination from where you stand to the top of this lighthouse. We have the reference angle, the side opposite the reference angle, and we are looking for the side adjacent to the reference angle. This is the tangent ratio: the side opposite the reference angle over the side adjacent to the reference angle. We know the measure of the reference angle and we know the value of the side opposite the reference angle. Filling in:

[tex]tan(2)=\frac{200}{x}[/tex] and

[tex]x=\frac{200}{tan(2)}[/tex] so

x = 5727.25 feet, the third choice down

Answer 2

Answer:

[tex]tan2=200/x[/tex] [tex]x=200/tan2[/tex]

[tex]x=5,727.25[/tex]

------------------------

hope it helps..

have a great day!!


Related Questions

What’s the answers???

Answers

Step-by-step explanation:

[tex]3)b \: c \ \: d \: e \\ \\ 4)bc \: nd \: de \\ 5)s \\ 6)t \\ thank \: you[/tex]

how many weeks are there in 504 hours

Answers

Answer:

26297.4 weeks

Step-by-step explanation:

hope it will help u

please mark me brillient

Based only on the information given in the diagram, which congruence
theorems or postulates could be given as reasons why AABC= ALMN?
Check all that apply
O A. LL
O B. ASA
I C. LA
D. HL
E AAS

Answers

3 Answers:

Choice A.  LLChoice D.  HLChoice F.  SAS

==========================================================

Explanation:

Let's go through the answer choices one by one.

A) This can be used because LL = leg leg, and this means we have two pairs of congruent legs. Those pairs are AC = LN and CB = NM. The LL theorem only applies to right triangles.B) This cannot be used. We don't have info about two pairs of angles. We only know that one pair of angles are the same (those 90 degree angles). So we can't form the second "A" in "ASA". This idea will come up again in choice C and choice E.C) This cannot be used. Why not? Because the "A" of "LA" refers to "acute angle". But unfortunately we don't know anything about the acute angles (whether they are congruent or not). The LA theorem can only be applied to right triangles.D) This can be used. We can use the HL (hypotenuse leg) theorem because we see that AB = LM are the pair of congruent hypotenuses, and you can use any of the congruent leg pairs to form the L of HL. Similar to LL and LA, the HL theorem only works for right triangles.E) This cannot be used. Like with choice B, we can't form the second "A" of "AAS".F) This can be used because we have two pairs of congruent sides, with a pair of congruent angles between those sides. Those angles being the marked 90 degree angles. It turns out that LL theorem is a special case of the SAS theorem.

In short, we can use choice A, choice D, choice F. We can't use the other three choices because we lack the info about any other pairs of angles.

The congruence theorem or postulate that we can use to show that triangle ABC is congruent to triangle LMN is LL (Side-Side-Side), the correct option is A.

What are congruent triangles?

Suppose it is given that two triangles ΔABC ≅ ΔDEF

Then that means ΔABC and ΔDEF are congruent. Congruent triangles are exact same triangles, but they might be placed at different positions.

The order in which the congruency is written matters.

For ΔABC ≅ ΔDEF, we have all of their corresponding elements like angle and sides congruent.

Thus, we get:

[tex]\rm m\angle A = m\angle D \: or \: \: \angle A \cong \angle D \angle B = \angle E\\\\\rm m\angle B = m\angle E \: or \: \: \angle B \cong \angle E \\\\\rm m\angle C = m\angle F \: or \: \: \angle C \cong \angle F \\\\\rm |AB| = |DE| \: \: or \: \: AB \cong DE\\\\\rm |AC| = |DF| \: \: or \: \: AC \cong DF\\\\\rm |BC| = |EF| \: \: or \: \: BC \cong EF[/tex]

(|AB| denotes length of line segment AB, and so on for others).

We are given that;

Sides are equal

Now,

Based only on the information given in the diagram, we can use the following congruence theorems or postulates to show that triangle ABC is congruent to triangle LMN:

A. LL (Side-Side-Side): This theorem states that if three sides of one triangle are congruent to three sides of another triangle, then the triangles are congruent. In this case, we know that AB = LM, AC = LN, and BC = MN, so we can use LL to show that triangle ABC is congruent to triangle LMN.

B. ASA (Angle-Side-Angle): This theorem states that if two angles and the included side of one triangle are congruent to two angles and the included side of another triangle, then the triangles are congruent. In this case, we do not know any angle measures, so we cannot use ASA to show that the triangles are congruent.

Therefore, by the  congruent triangles the answer will be LL (Side-Side-Side).

Learn more about congruent triangles here:

https://brainly.com/question/16921692

#SPJ7

please help in indices
[tex] \frac{ {5}^{m + 2} - {5}^{m} }{ {5}^{m + 1} + {5}^{m} } \\ \\ \frac{ {4}^{m} + {4}^{m + 1} }{ {4}^{m + 2} - {4}^{m} } [/tex]

Answers

Step-by-step explanation:

Hey there!

Please see your required answer in picture.

Hope it helps!

Please find attached photograph for your answer.

Hope it helps.

Do comment if you have any query.

I have 5 digits.My eight is worse 8000. One of my sixes is worth 60.The other is worth 10 times as much.My other digit is a zero

Answers

Answer:

i think its 58,660

Step-by-step explanation:

Answer:

58,660

Step-by-step explanation:

quadrilateral abcd is symmetric with respect to the y axis. the coordinates of point a are (-2,2), and the coordinates of point c are (2, 1). if b is in the first quadrant, what are the coordinates of b?​

Answers

Answer:

The coordinate of b = (2, 2)

Step-by-step explanation:

The details of the quadrilateral abcd are;

The quadrilateral is symmetric about the y-axis (the line x = 0)

The coordinates of the vertices are; a(-2, 2), c(2, 1)

The location of the vertex, b = The first quadrant

We have;

The line [tex]\overline {ab}[/tex] is perpendicular to the line [tex]\overline {bc}[/tex]

Let (x, y) represent the coordinate of the vertex b, we have;

(y - 2)/(x - (-2)) = (y - 2)/(x + 2) = -1/(y - 1)/(x - 2) = - (x - 2)/(y - 1)

(y - 2)·(y - 1) = -(x + 2)·(x - 2)

y² + x² - 3·y - 2 = 0...(1)

y² + x² = 3·y + 2

Also we have;

(y - 2)² + (x - 1)² + (y - 2)² + (x - (-2))² = (2 - 1)² + (2 - (-2))² = 17

Therefore;

2·y² + 2·x² - 8·y + 2·x + 13 = 17

2·(3·y + 2) - 8·y + 2·x + 13 = 17

Using an online tool, we have;

x = y

From equation (1), we have;

2·y² - 3·y - 2 = 0

∴ y = 2, or y = -1/2

Where y = 2, we have;

x = y = 2

Therefore, the point b = (2, 2).

Find the surface area of each figure. Round your answers to the nearest tenth, if necessary​

Answers

Answer: 109m^2

Step-by-step explanation:

(5)(8.4)/2 = 21 m^2

(4 ) (21) = 84 m^2

Base = 5^2 = 25 m^2

84 + 25 = 109 m^2

write as a sentence in words 7.8 > 3.4

Answers

Answer:

seven and eight tenths is greater than three and four tenths

Hope it helped you

Please help me with this anyone

Answers

Answer:

Step-by-step explanation:

Begin by combining like terms and then factoring. Combining like terms will give you

[tex]10p^2-17p-20=0[/tex] Using the "old-fashioned" way of factoring, the a times c method, our a = 10, b = -17 and c = -20.

a * c = 10(-20) = -200 and now we need the factors of 200 (don't worry about the negative) that combine to give us that middle term, -17p (here is where the negative matters). The factors of 200 are:

1. 200;  2, 100;  4. 50;  5, 40;  8, 25;  10, 20

The combination of those numbers that can be manipulated to give us a -17p is the 8, 25 as long as we say that the 25 is negative and the 8 is positive. Rewrite the original polynomial to reflect those factors:

[tex]10p^2-25p+8p-20=0[/tex] and then factor by grouping:

[tex](10p^2-25p)+(8p-20)=0[/tex] and factor out from each set of parenthesis what is common:

[tex]5p(2p-5)+4(2p-5)=0[/tex] again factor out what is common:

(2p - 5)(5p+ 4) = 0. These are the factors; therefore the solutions are

2p - 5 = 0 so

2p = 5 and

p = 5/2  and

5p + 4 = 0 and

5p = -4 so

p = -4/5

anyone help me. ....​

Answers

Answer:

see explanation

Step-by-step explanation:

Using the rules of exponents

All the exponents inside the parenthesis are multiplied by the exponent outside the parenthesis.

Then

[tex]\frac{m^{-7}n^{14} }{m^{-25}n^{40} }[/tex] ÷ [tex]\frac{m^{-21}n^{42} }{m^{-25}n^{40} }[/tex]

[ Using the rule [tex]\frac{a^{m} }{a^{n} }[/tex] = [tex]a^{(m-n)}[/tex] ]

= [tex]m^{(-7-14)}[/tex] [tex]n^{(14-(-28)}[/tex] ÷ [tex]m^{(-15-10)}[/tex] [tex]n^{(25-(-15)}[/tex]

= [tex]m^{-21}[/tex] [tex]n^{14+28}[/tex] ÷ [tex]m^{-25}[/tex] [tex]n^{25+15}[/tex]

= [tex]\frac{m^{-21}n^{42} }{m^{-25}n^{40} }[/tex]

= [tex]m^{(-21-(-25))}[/tex] [tex]n^{(42-40)}[/tex]

= [tex]m^{(-21+25)}[/tex] n²

= [tex]m^{4}[/tex] n² = [tex]m^{x}[/tex] [tex]n^{y}[/tex] , so x = 4 and y = 2

Then

[tex]m^{x-2y}[/tex] = [tex]m^{4-2(2)}[/tex] = [tex]m^{4-4}[/tex] = [tex]m^{0}[/tex] = 1 ← as required

What is the volume of the cylinder?
•576 cm3
•2887 cm3
•96 cm3
•192 cm3

Answers

[tex] \large\begin{gathered} {\underline{\boxed{ \rm {\red{Volume \: \: of \: \: cylinder \: = \: \pi \: {r}^{2} \: h }}}}}\end{gathered}[/tex]

r represents radius of cylinder.

h represents height of cylinder.

So ,

r = 6 cm

h = 16 cm

π = 3.14

Substuting the values

⇥Volume of the cylinder = π r² h

⇥Volume of the cylinder = 3.14 × 36 × 16

⇥Volume of the cylinder = 113.04 × 16

⇥Volume of the cylinder = 1808.64

Hence , the volume of cylinder is 1808.64 cm²

The breadth of a rectangle is 4 units less than its length. If the perimeter of the rectangle is 20 units, write a pair of linear equations to model the above situation, assuming the length to be l units and the breadth to be b units.

Answers

Answer:

L=7 units and B=3 units

Step-by-step explanation:

L=B+4 ... Equation 1

P = 20

P=2(B+L)... Equation 2

20=2(B+B+4)... Substituting equation 1 to equation 2

20=2(2B+4)

20=4B+8

20-8=4B

12=4B

12/4=4B/4

B=3... Solve for B

L=B+4

L=3+4

L=7... Solve for L

A. 4√29/ 7 feet
b. 4√5 feet
c.√164 feet
d. 2√41

Answers

Answer:

These two triangles are similar triangles. This means that their side lengths are proportional to each other.

Thus, making line segment EC equal to "x", and BC equal to "y" we can write:

8/y = 28/(10+y)

The next step is to get rid of the fractions, which can be done by cross multiplying.

So we have:

8(10+y) = 28(y)

After distribution and some simplification, you should get the value of y.

80+8y = 28y

80 = 20y

80/20 = 20y/20

4 = y

y = 4

Knowing that y = BC, and y = 4, it is clear that BC = 4.

Since BC = 4, one can use the Pythagorean Theorem to solve for segment EC.

Pythagorean Theorem: a^2 + b^2 = c^2, where a and b are the side lengths of a right triangle, and c is the hypotenuse (in other words the longest side)

In our case, a and  b are 8 and 4 (the order doesn't really matter here).

So we have: 8^2 + 4^2 = c^2

64 + 16 = c^2

80 = c^2

c = sqrt 80

c  = 4 sqrt 5

And we arrive at the answer- EC = 4 sqrt 5, making B the correct choice.

Hope this helps!

(4a)^2 without the exponents

Answers

Answer:

16 a*a

Step-by-step explanation:

(4a)^2

(4a) (4a)

16 a*a

Answer:

16a²

Step-by-step explanation:

(4a)²

=>(4)² x (a)²

=> 16 x a²

=>16a²

Y’all plss plss help me, it super ugent, I’ll mark the brainlest
Plsssssss, it Imp not spamming,, I’m serious I’ll report u

Answers

Answer:

the answer is correct

Step-by-step explanation:

hope u like

Please help me find which expression is correct

Answers

Answer:

IN MY OPINION D NO IS THE CORRECT ANSWER OF YOUR QUESTION.

Answer:

in my opinion d is the correct answer of your questions.

Step-by-step explanation:

Because subtract subtract sign is always add.

hope this will help you

thanks

can someone help me with this one ....​

Answers

Answer:

-5, - 2, 3

Step-by-step explanation:

y=2x+3, y=-7, x=-5; y=-1, x=-2, y=9, x=3

Enter a recursive rule for the geometric sequence?

4, -16, 64, -256,...

a^1 = __; a^n = __


Plzz help!

Answers

Answer:

Step-by-step explanation:

a^1=-2

a^n=a×a^n-1

what is the gcf of 36, 126, and 210?

Answers

Answer:

Greatest common factor (GCF) of 36 and 210 is 6.

Step-by-step explanation:

Answer:

6

Brainliest, please! (Almost an Ace!)

Step-by-step explanation:

Look at the smallest factor, 36. Find its factors.

36: 1, 2, 3, 6, 12, 18, 36

Out of all of them, which is the biggest one that 126 and 210 are also divisible by?

We see that they're divisible by 1, 2, 3, and 6.

Our answer is 6.

Find the least number by which the following number must be multiplied so that the product are perfect cube one number 72 to number 128 number 288 phone number 675​

Answers

Answer:

Well this question is actually a piece of cake. Just pick your favorite number. Multiply it by 10. Then do whatever operation you want with the 2,300. For the exponent part of this. Lets say we do it this way y times z equals 2,300. Exponents are letters used in mathematical terms. So any letter can be used to represent any number.

Step-by-step explanation:

With the given information, which of these conclusions is reasonable? A.Town A and Town B had the same low temperatures. B.The temperatures in each town were the same every day. C.The mean low temperatures in Town A and Town B were the same. D.The variability of temperatures in Town A and Town B was the same.

Answers

Answer:

A. Town A and Town B had the same low temperatures.

Step-by-step explanation:

The towns both A and B had low temperatures is shown by the same rate of low temperatures.

Plz help guys, it is a pretty easy sum​

Answers

Answer:

a3+13a

Step-by-step explanation:

sorry I don't know how to solve this

Answer:+-3

Explanation:

I added 4 on both side so that a^2+1/a^2-2 becomes a^2+1/a^2+2 which is Special products of (a^2+1/a^2).

should the quadratic formula be used to solve this question explain.


x^2+4x-3=0

Answers

Answer:

Yes

Step-by-step explanation:

This equation does not factorise, so quadratic equation will be useful. It is easier than other methods that can be applied to this equation.

Which is the best estimate of -14 1/9 (-2 9/10)

Answers

Answer:

Step-by-step explanation:

-14 1/9 is close to - 14

-2 and 9/10 is close to - 3

The best estimate would be 42 (14 * 3)

Estimate means you put your calculator on the kitchen counter until you've done this question.

Let's see what the actual answer is. 40.92222 which is close to 41.

42 is a pretty good estimate.

what is the exponents

Answers

The exponent of a number says how many times to use the number in a multiplication.

For Example, In 8² , the 2 is the number of times 8 has to be multiplied. In this case It has to multiplied by itself 2 times, 8x8 = 64 so 8² = 64.
Another example is 9³ since the number above is 3 we have to multiply 9 three times, 9x9x9 = 729 which means 9³ = 729.

Hope this helps

Plz solve question 12

Answers

Answer: C

Step-by-step explanation:

To solve for 12, we can use eliminate or substituion to solve our system of equations. Let's use elimination method.

[tex]\left \{ {{3f-2k=10} \atop {-3f-2k=14}} \right.[/tex]

Let's add the equations together. This way, 3f+(-3f)=0

[tex]-4k=24[/tex]                   [divide both sides by -4]

[tex]k=-6[/tex]

Now that we know k, we can plug it into either equation to find f.

[tex]3f-2(-6)=10[/tex]        [multiply]

[tex]3f+12=10[/tex]             [subtract both sides by 12]

[tex]3f=-2[/tex]                    [divide both sides by 3]

[tex]f=-\frac{2}{3}[/tex]

Now that we have f and k, we know that C is the correct answer.

Problem 3 Find the value of x.​

Answers

value of x should be 3.2 units

Answered by Gauthmath must click thanks and mark brainliest

3.2 units in your answer

Jimmy is saving money to buy a concert ticket for $155. He has $42 so far and he can save $20 per week. In how many weeks will he have enough money to buy the ticket? Select an equation that could be used to answer the question above. Let W represent the number of weeks. A. 20w + 42 = 155 B. 20w - 155 =42 C. 42w + 20 = 155 D. 155w - 20 = 42

Answers

Answer:

The answer for this question is A

6y−3=57 what does y equal

Answers

Answer:

y = 10

Step-by-step explanation:

6y−3=57

6y = 60

y = 10

Explain the process due to which rain falls ? Class 4 - EVS​

Answers

Answer:

As you may already know, water drops that fall from the cloud are considered "rain".

The Sun's heat turns the moisture or water from leaves, plants, rivers, lakes, and oceans - and turns it into gas or also called, vapor. This water vapor then turns into gas and disappears into the air. When it gets mixed with the air, it cools down. When it cools down, it changes into small water drops, which then form a cloud. These small water drops join together with other water drops to create larger and bigger water drops.

You may know this now because this is the easiest part. What happens when something gets heavy or is over-filled? It falls down, right?

So, the large drops from water fall down as they get too heavy for the cloud to carry. These big droplets falling down on us are called Rain.

Other Questions
1) The position of an object to the north of a flagpole is given by x(t) = bt2 c , where b and c are constants. a) What is v(t), the velocity of the object as a function of time? b) What is a(t), the acceleration of the object as a function of time? c) At some time t the object is located at the flagpole. What is the velocity of the object at that instant? If f(x) = x3 1, what is the ordered pair for x = -2.? What is the value of this expression when a = 7 and b = -4?|2a|-b over 3 Edgar accumulated $5,000 in loan debt. If the interest rate is 20% per year and he does not make any payments for 2 years, how much will he owe on this debt in 2 years for quarterly compounding? Round your answer to the nearest cent Do NOT round until you calculate the final answer. strontium-90 is a radioactive material that decays according to the function A(t)=A0e0.0244t, where A0 is the initial amount present and A is the amount present at time t (in years). Assume that a scientist has a sample of 400 grams of strontium-90. (a) What is the decay rate of strontium-90? (b) How much strontium-90 is left after 30 years? (c) When will only 100 grams of strontium-90 be left? (d) What is the half-life of strontium-90? (a) The decay rate of strontium-90 is nothing%. (Type an integer or a decimal. Include the negative sign for the decay rate.) 1. PLEASE HELP MEFind the value of x. If necessary, round to the nearest tenth.A. 8.8 in.B. 10.4 in.C. 5.2 in.D. 7.3 in. 3x?3x?If Ax)=2xand g(x)find /(x) = g(x) please help! need answers in order to move on:)1.) which number equals (5)^-3?- -125- 1/15- 1/125- -152.) find the equivalent for -(3)^-4- -(4x4x4)- -3x-4- 1/-3x-3x-3x-3- -(1/3x1/3x1/3x1/3)3.) which of the following is equivalent to 3^-8x3^4- 3^-12- 3^-4- 3^-2- 3^-324.) which value is equivalent to 7^-3/7^-5- 7^15- 7^-2- 7^8- 7^25.) choose the equivalent expression (8^10)^2- 8^12- 8^20- 8^8- 8^56.) which expression is equivalent to (9x8)^4- 9x8x4- 9^4x8^4- (9^8)^4- (9^4)^87.) which expression is equivalent to (2/7)^5- 2^5/7^5- 2x5/7x5- 2^5/7- 2x5/7 Which is the graph of y = [x]-2? Please please helped timed 20points Mrs. Turner finally rose to go after being very firm about several other viewpoints of either herself, her son or her brother. She begged Janie to drop in on her anytime, but never once mentioning Tea Cake. Finally she was gone and Janie hurried to her kitchen to put on supper and found Tea Cake sitting in there with his head between his hands. Which best describes the language in this excerpt Find 4^3(5^2). Express using exponents. I NEED HELP ON THIS AS FAST AS POSSIBLE List two important groups currently living in New Mexico. Then, briefly evaluate each groupto identify one way that the a group reflects a historical New Mexican influence. I need the answer of the question without date or percentage thank you guys so much What are the values of a, b, and c in the quadratic equation 0 = one-halfx2 3x 2?a = one-half, b = 3, c = 2a = one-half, b = 3, c = 2a = one-half, b = 3, c = 2a = one-half, b = 3, c = 2 What is the additive identity of -17? Harn una lista de programas de televisin que puedan ayudarnos en algunos de nuestros cursos escolares Larry made 14 baskets out of 21 attempts in a recent basketball game. If Scott attempted 24 baskets and made the same proportion of baskets as Larry, how many baskets did Scott make?Scott made baskets. WHO IS GREATER THAN JHON THE BAPTIST AND WHAT VERCE HELPPPPPP How is the function of the judicial branch most related to the function of the legislative branch?A. The judicial branch proposes new legislation to improve the state and local court systems.B. The judicial branch determines whether the laws passed by Congress are constitutional.C. The judicial branch oversees the impeachment of presidents and other government officials.D.The judicial branch settles disagreements between the Senate and the House of Representatives. how do you format a cd or a DVD A 21-year-old woman presents to the emergency department with fevers, headache, neck stiffness, and mild confusion over the past several days. Her temperature is 38.0 C (100.4 F), pulse 106, and blood pressure 116/74. On physical exam she looks ill, and her neck is stiff. Her neurologic exam is normal. A lumbar puncture reveals 105 WBC and 1240 RBC in tube #1 and 126 WBC and 1360 RBC in tube #4; all white cells are lymphocytes. The CSF protein is 68 and the glucose is 78. This patient most likely has which of the following?1.HSV encephalitis2.Pneumococcal meningitis3.Subarachnoid hemorrhage4.Subdural hematoma